Which function is a quadratic function?
O p(x) = 2x(x2 + 6) + 1
m(x) = -4(x + 3) - 2
Ot(x) = -8x²(x2 – 6) + 1
Oh(x) = 3x(x - 2) - 4

Answers

Answer 1
The answer is the fourth choice

Related Questions

PLEASE HELP, WILL GIVE BRAINLIEST FOR RIGHT ANSWER!
Indicate the equation of the given line in standard form, writing the answer in the equation box below.

The line that contains the point Q (1, -2) and is parallel to the line whose equation is
y − 4 = 2/3 ( x − 3)

Answers

Find gradient by changing form of equation: y = 2/3x + 4. Gradient is 2/3.

Find y intercept of other line by substituting the coordinates: -8/3

Use information to write out equation of line: y= 2/3x - 8/3

Convert to standard form: -2x + 3y = 8.

Hope this helps :)

Answer:

my previous account deleted...how r u sadie??

The value of the expression when x equals 12/7 is..

Answers

Answer:

x = 82

Step-by-step explanation:

1/3(12/7 + 9/7) + 3⁴ =

1/3(21/7) + 3⁴ =

1/3(3) + 3⁴ =

1 + 81 = 82

Express in simplest form -45%​

Answers

Answer: hello, your answer is -0.45

What is the sum of the fractions? Use the number line to help find the answer.|
+
5

Answers

Answer:

-4/5

Step-by-step explanation:

If you use the number line, after adding 3/5 you can see that it still doesn't make it positive but brings it up to -4/5 (Hope this helps)

Answer:

You subtract them:

3/5-7/5 (the plus disappears when faced with a minus)

-4/5

What is 2x/3 - 4 =x ?

Answers

Step-by-step explanation:

2x/3 -4 = x

multiply both side by 3

2x - 12 = 3x

-x = 12

x = -12

Calculate the length of line segment XY in each triangle.

Please No Docs/No File Only real answer
25 points and a 5 star rated

Answers

Answer:

XY = 10 cm

Step-by-step explanation:

XY is a midsegment and is half the length of the 3rd side DE, that is

XY = [tex]\frac{1}{2}[/tex] DE = [tex]\frac{1}{2}[/tex] × 20 = 10 cm

The graph shown is a scatter plot:
Which point on the scatter plot is an outlier?

Answers

Answer:

D is the outlier

Step-by-step explanation:

An outlier is a point that is far from the other points

We can draw a line that roughly represents an equation for the points

A,B ,C are all near the line

D is not along the line

Which function is positive for the entire interval [–3, –2]?

On a coordinate plane, a curved line with a minimum value of (0, negative 3) crosses the x-axis at (negative 3, 0) and (3, 0), and crosses the y-axis at (0, negative 3).

On a coordinate plane, a curved line with a minimum value of (2, negative 3) crosses the x-axis at (negative 1, 0) and (5, 0), and crosses the y-axis at (0, negative 1.5).

On a coordinate plane, a curved line with a minimum value of (2, 4) and a maximum value of (0.5, 6), crosses the x-axis at (negative 1.5, 0) and crosses the y-axis at (0, 5).

On a coordinate plane, a curved line with a minimum value of (negative 1.75, negative 3.9) and a maximum value of (0, 2), crosses the x-axis at (negative 2.2, 0), (negative 0.75, 0), and (0.75, 0), and crosses the y-axis at (0, 2).

Answers

Answer:

B

Step-by-step explanation:

Based on the given information, option 4 seems to be the most likely candidate to be positive for the entire interval [-3, -2]. Correct option is 4.

To determine which function is positive for the entire interval [-3, -2], we need to look for the part of the curve that lies above the x-axis within this interval.

The curve has a minimum value of (0, -3) and crosses the x-axis at (-3, 0) and (3, 0), which means it goes below the x-axis in the interval [-3, -2]. So, this option is not positive for the entire interval.

The curve has a minimum value of (2, -3) and crosses the x-axis at (-1, 0) and (5, 0). Since it crosses the x-axis at -1 which is outside the interval [-3, -2], this option is not positive for the entire interval.

The curve has a minimum value of (2, 4) and a maximum value of (0.5, 6). Since the minimum value is above the x-axis, this option is not positive for the entire interval.

The curve has a minimum value of (-1.75, -3.9) and a maximum value of (0, 2). It also crosses the x-axis at (-2.2, 0), (-0.75, 0), and (0.75, 0). Since the minimum value is above the x-axis, and all the points where it crosses the x-axis are outside the interval [-3, -2], this option could potentially be positive for the entire interval.

To know more about interval:

https://brainly.com/question/34520204

#SPJ3

supplementary angles measure _____ degrees?​

Answers

my answer

90°to 180°degrees

PLEASE HELP! What is the number ( code ) ?

Answers

Answer:

The number code is: 12.95

Step-by-step explanation:

98.86 + 74.57 = 173.43

345.32 - 19.78 = 325.54

0.79 x 0.86 = 0.6794

5733.75 + 75 = 5,808.75

So, if you put the numbers together you will get -

12.95

Hope this helps!

Good Luck!

:)

If you can, please mark me brainiest

12.95
explanation
i’ve don’t this before

Does this graph represent a function? Why or why not?

Answers

Answer:

B yes, because it pases the vertical line test

A group of students were surveyed to find out if they like playing tennis or volleyball in PE class. The results are shown in the table:
Complete the table and answer the following questions.

a. What percentage of the total students surveyed like both
Volleyball and Tennis?

b. What is the probability that a student who likes
Volleyball does not like Tennis?

Answers

Answer:

a) 41.666...%

b) 10/120 or 1/12

Step-by-step explanation:

50 students like volleyball and tennis out of 120 students surveyed. 50/120 as a percent is 41.6666666...%

10 students like volleyball but not tennis out of 120 students surveyed. 10/120 can also simplify to 1/12

:) ur welcome

Write an equation that PASSES THROUGH (3, -1) and is PARALELL to y = 6x - 4
show work

Answers

Answer:

y = 6x - 19

Step-by-step explanation:

The equation of a line in slope- intercept form is

y = mx + c ( m is the slope and c the y- intercept )

y = 6x - 4 ← is in slope- intercept form

with slope m = 6

Parallel lines have equal slopes , then

y = 6x + c ← is the partial equation

To find c substitute (3, - 1) into the partial equation

- 1 = 18 + c ⇒ c = - 1 - 18 = - 19

y = 6x - 19 ← equation of parallel line

What decimal is equivalent to 1/8

Answers

Answer:

0.125

Step-by-step explanation:

Becuase math

Put these numbers in order from least to greatest.

0.1, 1/2, and 0.2

Answers

Answer:

0.1, 0.2, 1/2

Answer:

0.1<0.2<0.5(1/2)

hope it helps

have a nice day

Worth 10 pts
Plssssssssssssssssss help
Will give brainliest

Answers

Answer:  [tex]2\sqrt{2}+\sqrt{3}\\\\[/tex]

a = 2 and b = 1

=======================================================

Explanation:

Set the expression equal to the given form we want. Then square both sides so we get rid of the outer-most square root

[tex]\sqrt{11+4\sqrt{6}} = a\sqrt{2}+b\sqrt{3}\\\\\left(\sqrt{11+4\sqrt{6}}\right)^2 = \left(a\sqrt{2}+b\sqrt{3}\right)^2\\\\11+4\sqrt{6} = \left(a\sqrt{2}\right)^2+2*a\sqrt{2}*b\sqrt{3}+\left(b\sqrt{3}\right)^2\\\\11+4\sqrt{6} = 2a^2+2ab\sqrt{2*3}+3b^2\\\\11+4\sqrt{6} = 2a^2+3b^2+2ab\sqrt{6}\\\\[/tex]

In the third line, I used the rule that (x+y)^2 = x^2+2xy+y^2

-------------------

At this point, we equate the non-radical and radical terms to get this system of equations

[tex]\begin{cases}11 = 2a^2+3b^2\\ 4\sqrt{6} = 2ab\sqrt{6}\end{cases}[/tex]

The second equation turns into 4 = 2ab when we divide both sides by sqrt(6)

Then 4 = 2ab turns into ab = 2 after dividing both sides by 2.

We're told that a,b are rational numbers. Let's assume that they are integers (which is a subset of the rational numbers).

If so, then we have these four possibilities

a = 2, b = 1a = -2, b = -1a = 1, b = 2a = -1, b = -2

If a,b are negative, then you'll find that [tex]a\sqrt{2}+b\sqrt{3}[/tex] overall is negative. But this contradicts that [tex]\sqrt{11+4\sqrt{6}}[/tex] is positive. So a,b must be positive.

Let's assume that a = 1 and b = 2. If so, then,

2a^2+3b^2 = 2(1)^2+3(2)^2 = 14

but we want that result to be 11 instead.

Let's try a = 2 and b = 1

2a^2+3b^2 = 2(2)^2+3(1)^2 = 11

which works out perfectly.

Therefore,

[tex]\sqrt{11+4\sqrt{6}} = 2\sqrt{2}+\sqrt{3}\\\\[/tex]

---------------------------------

Checking the answer:

Use a calculator to find that

[tex]\sqrt{11+4\sqrt{6}} \approx 4.5604779\\\\2\sqrt{2}+\sqrt{3} \approx 4.5604779\\\\[/tex]

both have the same decimal approximation, so this is a fairly informal way to confirm the answer.

Another thing you can do is to take advantage of the idea that if x = y, then x-y = 0

So if you want to see if two things are equal, you subtract them. You should get exactly 0 or something very small (pretty much equal to 0).

13 subtracted from a number is at most 10

Answers

Step-by-step explanation:

[tex]13 - x \leqslant 10[/tex]

[tex]3 \leqslant x[/tex]

What is the slope of the line represented by the equation y = -1/2x + 1/4

Answers

Answer:

The slope is -1/2 and the y intercept is 1/4

Step-by-step explanation:

y = -1/2x + 1/4

This equation is in slope intercept form

y = mx+b where m is the slope and the y intercept is b

The slope is -1/2 and the y intercept is 1/4

Pls Solve

2/5 × (-3/7) - 1/6 × 3/2 +1/14 × 2/5


Answers

Answer:

-11/28

Step-by-step explanation:

Following BODMAS:

(2/5 * -3/7) - (1/6 * 3/2) + (1/14 * 2/5)

= (-6/35) - (1/4) + (1/35)

= 1/35 - 6/35 - 1/4

= -5/35 -1/4

= -55/140

= -11/28

A rectangle has a length that is 8 less than it’s width w the perimeter is 52 which equation can be used to determine length

Answers

Answer:

width?

8-W-52

2(w-8) +2w=52

(W-8)+W=52

2(8-W) +2w=52

is (6,3) a solution to this system of equations y=3x - 3 3x - y =3

Answers

Answer:

It is not a solution

Step-by-step explanation:

Plug the point into the equations and check to see if they are true

y=3x - 3

3 = 3(6) -3

3 = 18-3

3 = 15

False

We do not need to check the other equation since this is false

Which expression is equivalent to 24x2 - 22x + 5?​

Answers

Answer:

C

Step-by-step explanation:

Given

24x² - 22x + 5

Consider the factors of the product of the x² term and the constant term which sum to give the coefficient of the x- term.

product = 24 × 5 = 120 and sum = - 22

The factors are - 12 and - 10

Use these factors to split the x- term

24x² - 12x - 10x + 5 ( factor the first/second and third/fourth terms )

= 12x(2x - 1) + 5(2x - 1)  ← factor out (2x - 1) from each term

= (12x - 5)(2x - 1) ← in factored form → C

I’m begging you too pls pls pls pls help! I’d appreciate it so much. This is independent work and I have no clue of what this means so pls comment as fast as u can because according to my teacher this was supposed to be due yesterday and I’m just doing it today. Pls help thank you so so so much! Pls don’t waste time thanks again!

Answers

1) 1 hour and 15 minutes

2) 4:07

3) 30 minutes

4) 8:35

9) 1 hour and 53 minutes

10) 4:17

11) 2 hours and 52 minutes

12) 3:35

13) 9:55

14) 8:10

15) 1 hour and 45 minutes

16) 3:05

17) 6:48

18) 9:45

19) 1 hour and 25 minutes

20) 2 hours and 45 minutes

Brainliest would be appreciated :)

please help me solve this problem one problem for 15 points

Answers

i didn't get it what is the question

the perimetee are 81cm and the ratio sides are 2:3:4 find the area of triangle​

Answers

Answer: 18,27,36

Step-by-step explanation: We have to add all the ratios to get 9 and then divide 81 by 9 and get 9. The multiple 2, 3 and 4 by 9 getting 18, 27 and 36 respectively.

1
Select the correct answer from each drop-down menu.
The function () = 13 has been transformed, resulting in function h.
h(t) = -(1 + 2)2 – 4
To create function h, function fwas translated 2 units
„. translated 4 units
and reflected across the

Answers

Answer:

To create function h, function f was translated 2 units right , translated 4 units up and reflected across the x axis

Step-by-step explanation:

Given

[tex]f(x) = x^3[/tex]

[tex]h(x) =-(x + 2)^3 - 4[/tex]

Required

Complete chart

First: f(x) was translated right by 2 units

The rule of right translation is [tex](x,y) \to (x + 2,y)[/tex]

So, we have:

[tex]f'(x) = f(x + 2)[/tex]

[tex]f'(x) = (x + 2)^3[/tex]

Next: f'(x) was translated up by 4 units

The rule of down translation is [tex](x,y) \to (x,y+4)[/tex]

So, we have:

[tex]f"(x) = f'(x) +4[/tex]

[tex]f"(x) = (x + 2)^3 +4[/tex]

Lastly, f"(x) was reflected across the x-axis;

The rule of this reflection is: [tex](x,y) \to (x,-y)[/tex]

So, we have:

[tex]h(x) = -f"(x)[/tex]

[tex]h(x) = -[(x+2)^3 + 4][/tex]

Remove bracket

[tex]h(x) = -(x+2)^3 - 4[/tex]

\sqrt(125)-\sqrt(45)+\sqrt(5)
ans=3\sqrt(5)

Answers

Answer:

what?

Step-by-step explanation:

........................

what is the nth term of 1/3 4/7 7/11 2/3 ​

Answers

Answer:

[tex]\frac{1+3(n-1)}{3+4(n-1)}[/tex]

Step-by-step explanation:

First find the pattern.

1/3 gets 5/21 added to it but doing that to 4/7 does not get 7/11, so it's nto normal adding.

if you multiply 1/3 gets 12/7, but if you multiply that by 4/7 you don't get 7/11 so it's not normal multiplcation.

I would next try only adding to the numerator and denominator separately.  

so 1/3 gets 3 added to the 1 and 4 added to the 3.  Doing that again gets us (4+3)/(7+4) = 7/11 and doing it again gets us 10/15 = 2/3.  So that is the right answer.

So we know what is happening.  if you start with 1/3 and increasingthe numerator by 3 and denominator by 4 then we know it's going to look like (1+3(n-1))/(3+4(n-1)) because the first term  is when n=1 and we want that to cancel out.  You can also simplify it and get (3n-2)/(4n-1)

Let me know if it doesn't make sense.  

Joe and Becky are buying fish for their new aquarium. The expression 3g + 4a represents the cost in dollars of buying fish
where 'g' is the number of guppies purchased and 'a' is the number of angel fish purchased. Find the cost of fish if they buy 12 guppies and 4 angel fish. Explain your work.

Answers

Answer:

$52

Step-by-step explanation:

Find the cost by plugging in 12 as g and 4 as a into the expression:

3g + 4a

3(12) + 4(4)

36 + 16

= 52

So, the cost will be $52

The sum of two numbers is 45. One number is 9 more than the
other one. Find the numbers.

Answers

Answer:

27+18=45

Step-by-step explanation:

x+y=45

x+9=y

(x+9)+x=45

(18+9)+18=45

27+18=45

The value of the two numbers 'x' and 'y' will be 18 and 27., respectively.

What is the solution to the equation?

The allocation of weights to the important variables that produce the calculation's optimum is referred to as a direct consequence.

The sum of the two numbers is 45. One number is 9 more than the other one.

Let the two numbers be 'x' and 'y'. Then the equations are given as,

y = x + 9              ...1

x + y = 45           ...2

From equations 1 and 2, then we have

x + x + 9 = 45

2x = 36

x = 18

The value of the variable 'y' is given as,

y = 18 + 9

y = 27

The value of the two numbers 'x' and 'y' will be 18 and 27., respectively.

More about the solution of the equation link is given below.

https://brainly.com/question/545403

#SPJ2

Other Questions
What is 2.3(x 2) = 15 healthy individuals are beneficial to a countrys development because such individualsare MORE likely to be(a) less critical (c) self sufficient and self reliant(b) satisfied and contented (d) productive and effective workers Analyze the diagram below and complete the statement that follows.The perimeter of the square isA. 42B. 60C. 110.25D. 112.5 Trevor heard a burglar entering through a living room window.He grinned as he picked up his gun. Crouching behind the sofa in his darkened home,he ambushed and killed the intruder with several well placed shots.He then added another notch in his trusty side-arm.Trevor most probably:____________ a. has exercised his constitutional right of self-defense. b. has acted legally,because the shooting took place inside his home. c. has acted legally if,but only if,the burglar was armed with a gun. d. is guilty of a homicide,or at least voluntary manslaughter. can someone help me???? 3.42x16.5 show your work plz For what values of b will F(x) = logo x be a decreasing function?A. b>0B. 0 C. b< 0D. O >b>-1 Which three factors determine the formality of a discussion?O opinion, audience, and timeO topic, audience, and purposeO facts, purpose, and locationO topic, time, and location Write a loop that continually asks the user what food the user has in their refrigerator until the user enters apples, in which case the loop ends. It should acknowledge the user in the following format. For the first food, the user might say "hamburger," so your response would be, "You have a hamburger with a total of 1 food(s) if they enter hamburger, and so on until they enter apples at which point the loop ends. ---- make sure you save your file as "may28.py" * In a carnival game, there are 8 identical boxes, one of which contains a prize. Contestants guess which box contains the prize. The game is played until one of the contestants guesses it correctly. A contestant with the smaller number of guesses wins the prize. Before each game, a new prize is placed at random in one of the 8 boxes.Requried:Is it appropriate to use the binomial probability distribution to find the probability that a contestant who plays the game that day several times wins exactly 4 times? **PLEASE HELP**The frequency table below represents the 30 best battling averages for a semi pro baseball league. Which ranges of battling averages were least common among the players XYZ uses a single plantwide predetermined overhead rate based on MHs. It estimated the following data: Total MHs 40,000 Total fixed MOH cost $ 344,000 Variable MOH per MH $ 3.90 Recently, Job M759 was completed. It required 60 MHs. The overhead applied to Job M759 is closest to:______ (Round your intermediate calculations to 2 decimal places.)a. $516 b. $234 c. $750 d. $984 Find the value of x to the nearest tenth. A. 26.3 B. 5.7 C. 9 D. 8.6 What is the perimeter of this rectangle? What is the mothers name in two kinds? what is a reflection of a story X+ 5If m(x) =x-1 and n(x) = x-3, which function has the same domain as (mon)(x)?X+5O (x)=1111o h(x)=X-111O (X)=X-411Oh(x) =X-3 Does anybody know the answer to this? Who was the first person to ever make a famous painting. Which of these was not one of the main causes of the devastating explosion in Chernobyl?A. radiationB. excess steamC. human errorD. hydrogen A scientist adds 5 mcg of the mineral cadmium to the daily diet of 100 4-week-old laboratory mice. After twelve weeks, the researcher weighs and takes blood samples from each mouse. According to her findings, the mice lost weight during the 12-week period and they have abnormal levels of certain hormones in their blood. Based on this information and your knowledge of scientific research, what would you tell the researcher about her findings?A. She needs to conduct more tests on the animals' blood, because her findings areincomplete.B. The findings are not meaningful or valid because of the way she designed her study.C. Her results are very interesting and she should report her findings to nutrition scientists, sothey can repeat her study and confirm the results.D. The findings need to be summarized in a research article for submission to a peer-reviewednutrition journal.